Problem

Source: USA TSTST 2013, Problem 9

Tags: geometry, rotation, invariant, trigonometry, inequalities, Niven theorem, geocombontturnedintoalg



Let $r$ be a rational number in the interval $[-1,1]$ and let $\theta = \cos^{-1} r$. Call a subset $S$ of the plane good if $S$ is unchanged upon rotation by $\theta$ around any point of $S$ (in both clockwise and counterclockwise directions). Determine all values of $r$ satisfying the following property: The midpoint of any two points in a good set also lies in the set.